Bedingte Wahrscheinlichkeit, Unfallbericht

Neue Frage »

Gast11022013 Auf diesen Beitrag antworten »
Bedingte Wahrscheinlichkeit, Unfallbericht
Meine Frage:
Hi,

wäre jemand so freundlich über meine Lösung zu folgender Aufgabe zu gucken?

Aus dem Unfallbericht einer Polizeidirektion werden folgende Ursachen am häufigsten angegeben. (Es sind 95% der Unfälle erfasst.)

A: Überhöhte Geschwindigkeit 45%
B: Nichtbeachten der Vorfahrt 25%
C: Trunkenheit am Steuer 15%
D: Zu geringer Abstand und sonstige Gründe 10%

Man setzt nun voraus, dass keine zwei Ursachen zur gleichen Zeit möglich sind.

Im Fall A liegt der entstandene Schaden in 75% der Fälle über 5000 Euro.
Im Fall B liegt der entstandene Schaden in 60% der Fälle über 5000 Euro.
Im Fall C liegt der entstandene Schaden in 30% der Fälle über 5000 Euro.
Im Fall D liegt der entstandene Schaden in 20% der Fälle über 5000 Euro.


a) Berechnen Sie den Anteil der Unfälle mit einer Schadenshöhe unter 5000 Euro.

b) Berechnen Sie die Wahrscheinlichkeit, dass Trunkenheit die Ursache war, bei einem Schaden über 5000 Euro.

c) Der Schaden liegt höher als 5000 Euro. Berechnen Sie die Wahrscheinlichkeit, dass Ursache D der Grund war.

Baumdiagramm ist unten angehängt.

Meine Ideen:
a) Ich habe die einzelnen Pfade, dass der Schaden unter 5000 Euro lag zusammen addiert:



b) Ich habe mit dem Satz von Bayes gerechnet:



Im Zähler steht 0,5525 wenn man es ausrechnet.

c) Ich gehe so vor wie in b)





Ich hoffe das Baumdiagramm ist deutlich genug:
Das was unten in blau steht muss an jedem einzelnem Pfad unten stehen. Das konnte ich aber aus Platz gründen nicht tuen.
Die Zahlen sind in Prozent angegeben.
Der Pfad wo 5 dran steht soll den Möglichkeit beziffern, dass der Unfall nicht aufgeführt wurde.
Von diesem Pfad gehen keine weiteren Äste ab.
Ich hoffe ihr erkennt die Aussage des Baumes.
opi Auf diesen Beitrag antworten »

verwirrt
In der Aufgabenstellung fehlt mir ein Wort:
Zitat:
a) Berechnen Sie den Anteil der erfassten Unfälle mit einer Schadenshöhe unter 5000 Euro.

Über die nichterfassten Unfälle sind keine Aussagen über die Schadenshöhe bekannt, also dürfen wir nur die Anteile der erfassten Unfälle berechnen. Entsprechend sähe der Rechenweg für a) dann aus.

Die Rechenwege für b) und c) können aber so bleiben, die Brüche (unter dem Bruchstrich steht übrigens der Nenner) kürzen sich weg.

Zitat:
Man setzt nun voraus, dass keine zwei Ursachen zur gleichen Zeit möglich sind.
Wenn ich mit dem Handy am Ohr besoffen über eine Kreuzung rase, mache ich alle Ursachen gleichzeitig möglich. geschockt
Gast11022013 Auf diesen Beitrag antworten »

Dann war es ja fast richtig.
Nur bei Aufgabe a) einen dummen Fehler gemacht.

Das mit dem Zähler weiß ich auch normalerweise, keine Ahnung warum ich mich vertan habe.

Ist den auch die Notation soweit in Ordnung?
Gerade beim Satz von Bayes?

Auf jeden Fall schon mal großen Dank, dass du dir die Zeit genommen hast es zu kontrollieren. Freude


Edit: Jetzt im nachhinein muss ich jedoch leicht an deiner Korrektur für Aufgabe a) zweifeln.

Die Wahrscheinlichkeit, dass z.B. jemand mit Überhöhter geschwindigkeit fährt wäre ja trotzdem 45%??

Edit2: Achso ich glaube ich habe es begriffen. Die Wahrscheinlichkeit, dass der Unfall erfasst wurde ist ja bedingt von der Wahrscheinlichkeit das er nicht erfasst wurde. Und deshalb sind es doch mit 95 statt 100 zu rechnen. Aber irgendwie bin ich nicht sicher.
opi Auf diesen Beitrag antworten »

Einen dummen Fehler hast Du sicher nicht gemacht, ich halte die Aufgabenstellung für ungenau.
Die Notation der bedingten WSK ist falsch: Bei Aufgabe b) z.B muß es heißen. Die WSK, daß C eintritt unter der Voraussetzung, daß >5000 eingetreten ist.

Zu Deinen Edits: Könnte auch mehr als 45% sein. Aber man weiß es ja nicht. Aus Edit 2 werde ich nicht schlau. Big Laugh
Gast11022013 Auf diesen Beitrag antworten »

Ist nicht so wichtig.
Ich denke ich habe begriffen weshalb ich von 95 ausgehen muss.

Danke für die Hilfe.
Ansonsten können wir das morgen bzw. heute Mittag noch klären wenn ich noch fragen habe.
Aber ich muss jetzt auch dringend ins Bett.
opi Auf diesen Beitrag antworten »

Dann wünsche ich Dir eine gute Nacht! Schläfer

Ich halte die Aufgabe für ungenau gestellt, hoffentlich irre ich ich nicht selbst.

Edit: Bitte leere Dein PN-Postfach, Cravour versucht vergeblich, Dir eine Nachricht zuzusenden. Nachfolgendes OT wie angekündigt entfernt.
 
 
Gast11022013 Auf diesen Beitrag antworten »

Laut der Lösung des Lehreres war dein Gedanke nicht ganz richtig.
Für Aufgabe a) war mein Lösungsweg korrekt.

Ist aber nicht so schlimm. Ich hatte es noch nicht verändert gehabt.

Wink
opi Auf diesen Beitrag antworten »

Danke für die Rückmeldung, ich habe es fast befürchtet.
Mit meinem Reparaturversuch der Aufgabenstellung wollte ich einen Antwortsatz wie "Der Anteil der Unfälle mit einer Schadenshöhe unter 5000 Euro beträgt 39,75%, es können aber auch bis zu 5% mehr sein" vermeiden. smile
Neue Frage »
Antworten »



Verwandte Themen

Die Beliebtesten »
Die Größten »
Die Neuesten »